Fourier series, complex coefficents

Click For Summary
The discussion revolves around finding a relationship between the complex Fourier coefficients of a periodic function f and a transformed function g, defined as g = e^{2it}f(t-3). It is established that both f and g are periodic with a period of 2π. The Fourier coefficients for f and g are expressed as c_n and d_m, respectively, with the transformation involving integration and shifting of the function f. The calculations lead to a representation of d_m in terms of c_n, specifically incorporating a phase shift due to the transformation. The thread seeks confirmation on the correctness of the derived relationship and invites further clarification if needed.
malawi_glenn
Science Advisor
Messages
6,735
Reaction score
2,434

Homework Statement



Let f be a periodic function with period 2pi

Let:

g = e^{2it}f(t-3)

Find a relation between f and g's complex Fourier coefficents.


Homework Equations



y(t) = \sum _{n-\infty}^{\infty} b_n e^{in\Omega t}

T \Omega = 2\pi

T is period

b_n = \dfrac{1}{T}\int _0^{T} y(t)e^{-in\Omega t}dt


The Attempt at a Solution



g is also periodic with 2pi, since e^{2it} is.

f(t) = \sum _{n = -\infty}^{\infty} c_n e^{in t}

c_n = \dfrac{1}{2\pi}\int _0^{2\pi} f(t)e^{-int}dt

g(t) = \sum _{m = -\infty}^{\infty} d_m e^{im t}

d_m= \dfrac{1}{2\pi}\int _0^{2\pi}e^{2it}e^{-imt}f(t-3)dt

f(t-3) = \sum _{n = -\infty}^{\infty} \left( \dfrac{1}{2\pi}\int _0^{2\pi} f(t-3)e^{-int}dt \right) e^{int}

d_m= \dfrac{1}{2\pi}\int _0^{2\pi}e^{it(2-m)}\left( \sum _{n = -\infty}^{\infty} \left( \dfrac{1}{2\pi}\int _0^{2\pi} f(t-3)e^{-int}dt \right) e^{int} \right) dt

r = t-3

Then r = t

d_m = \dfrac{1}{2\pi}\int _0^{2\pi}e^{it(2-m)}\left( \sum _{n = -\infty}^{\infty} \left( \dfrac{1}{2\pi}\int _{-3}^{2\pi -3} f(t)e^{-in(t+3)}dt \right) e^{int} \right) dt

We are still integrating over an integer multiple of periods, so that we can write:

d_m = \dfrac{1}{2\pi}\int _0^{2\pi}e^{it(2-m)}\left( \sum _{n = -\infty}^{\infty} \left( \dfrac{1}{2\pi}\int _{0}^{2\pi} f(t)e^{-int}e^{-i3n}dt \right) e^{int} \right) dt

d_m= \dfrac{1}{2\pi}\int _0^{2\pi}e^{it(2-m)}\left( \sum _{n = -\infty}^{\infty}c_n e^{-i2n}dt \right) dt

Does this look right to you guys? I don't have an answer to this one, and we have no smiliar problems in our course book =/
 
Last edited:
Physics news on Phys.org
Do you want more info?
 
Question: A clock's minute hand has length 4 and its hour hand has length 3. What is the distance between the tips at the moment when it is increasing most rapidly?(Putnam Exam Question) Answer: Making assumption that both the hands moves at constant angular velocities, the answer is ## \sqrt{7} .## But don't you think this assumption is somewhat doubtful and wrong?

Similar threads

  • · Replies 16 ·
Replies
16
Views
2K
Replies
1
Views
2K
  • · Replies 1 ·
Replies
1
Views
2K
  • · Replies 3 ·
Replies
3
Views
2K
  • · Replies 1 ·
Replies
1
Views
2K
  • · Replies 2 ·
Replies
2
Views
1K
Replies
2
Views
2K
  • · Replies 3 ·
Replies
3
Views
1K
  • · Replies 6 ·
Replies
6
Views
3K
  • · Replies 13 ·
Replies
13
Views
2K